Circuit with multiple batteries finding the current through each one

In summary, by setting up equations using Kirchhoff's rules, you were able to find two different currents (Current 1 and Current 3) in the circuit. However, you were unable to solve for these currents due to the cancellation of the common factor Current 3-1. This may indicate that there is a mistake in your setup of equations, or that further simplification is necessary.
  • #1
tmint
6
0
Diagram: http://gyazo.com/0b719e6d399ad894d2c48ce06d994672

Homework Statement


In the figure the resistances are R1 = 0.77 Ω and R2 = 1.8 Ω, and the ideal batteries have emfs ε1 = 2.1 V, and ε2 = ε3 = 3.5 V. What are the (a) current (in A) in battery 1, (b) the current (in A) in battery 2 and (c) the current (in A) in battery 3? (d) What is the potential difference Va - Vb (in V)?

Homework Equations


V=IR
Kirchoff's Loop Rules

The Attempt at a Solution


Okay, for some reason I solved a part of this or two before and hoped to get back to it.
For part (a), I set up the equation of I being the current and R being the resistance; B being battery:

I= B2-B1/(4R1+R2)

And I got the answer .287A and then multiplied that by 2 and got the first part which is .587A. I understand the logic of its the potential difference divided by the equivalent resistance but why is that multiplied by 2, any reason for that? I didn't understand.

For parts (b)+ (c), I tried to set it up in a similar way but they are both wrong:

For part (b): B2-B1/(4R1+R2) and I got .287A

For part (c): I did the same equation as above putting in B3 instead of B2 and got .287APart (d) I was able to do by finding the potential difference between points (a) and (b) which I took the .287A value that I calculated in (b) and (c) and set it up like this:

ΔV=-I2R2^2+(3.5V) which comes out to 2.98V which was correct. I could use some help with this problem, I understand how the equation to solve B is found out, but I don't understand why you would have the EQ resistance to find the first battery, rather than the resistance the battery goes through. Could use a nice explanation on that. Thanks!

If someone could help with doing it the Kirchhoff rule way, I tried to assign currents with directions but for some reason I get something completely different from the correct answer I got in part (a).
I have a test on this material Tuesday and I REALLY need to do well, I blanked out on the first exam and forgot everything and did really, really bad :uhh:.

I really appreciate it! Thank you!
 
Last edited:
Physics news on Phys.org
  • #2
tmint said:
Diagram: http://gyazo.com/0b719e6d399ad894d2c48ce06d994672

Homework Statement


In the figure the resistances are R1 = 0.77 Ω and R2 = 1.8 Ω, and the ideal batteries have emfs ε1 = 2.1 V, and ε2 = ε3 = 3.5 V. What are the (a) current (in A) in battery 1, (b) the current (in A) in battery 2 and (c) the current (in A) in battery 3? (d) What is the potential difference Va - Vb (in V)?

Homework Equations


V=IR
Kirchoff's Loop Rules

The Attempt at a Solution


Okay, for some reason I solved a part of this or two before and hoped to get back to it.
For part (a), I set up the equation of I being the current and R being the resistance; B being battery:

I= B2-B1/(4R1+R2)
Hello tmint. Welcome to PF !


I get something different for I1, the current through battery 1.

Show what equations you get from Kirchhoff's Laws.

(Also, your notation is a bit confusing.)

And I got the answer .287A and then multiplied that by 2 and got the first part which is .587A. I understand the logic of its the potential difference divided by the equivalent resistance but why is that multiplied by 2, any reason for that? I didn't understand.

For parts (b)+ (c), I tried to set it up in a similar way but they are both wrong:

For part (b): B2-B1/(4R1+R2) and I got .287A

For part (c): I did the same equation as above putting in B3 instead of B2 and got .287A


Part (d) I was able to do by finding the potential difference between points (a) and (b) which I took the .287A value that I calculated in (b) and (c) and set it up like this:

ΔV=-I2R2^2+(3.5V) which comes out to 2.98V which was correct.


I could use some help with this problem, I understand how the equation to solve B is found out, but I don't understand why you would have the EQ resistance to find the first battery, rather than the resistance the battery goes through. Could use a nice explanation on that. Thanks!

If someone could help with doing it the Kirchhoff rule way, I tried to assign currents with directions but for some reason I get something completely different from the correct answer I got in part (a).
I have a test on this material Tuesday and I REALLY need to do well, I blanked out on the first exam and forgot everything and did really, really bad :uhh:.

I really appreciate it! Thank you!
 

Attachments

  • 0b719e6d399ad894d2c48ce06d994672.jpg
    0b719e6d399ad894d2c48ce06d994672.jpg
    8.9 KB · Views: 1,241
  • #3
Thank you.

Sorry for the notation I'll write it out to make it easier to follow. When I use Kirchhoff's rules I get the following equations:
Let c be left of a, d left of b, f right of b, and e right of a

Oh, and Current 1 is current from Battery 1, Current 2 from Battery 2 in the middle branch and so on.

Current 3= Current 1+ Current 2

cabdc: -.77(Current 1)+1.8(Current 2)- 3.5-.77(Current 1)+2.1=0

aefba: -.77(Current 3)-3.5-.77(Current 3)+3.5-1.8(Current 2)

I use the first equation and solve for Current 2 since Current 2 is apart of both equations.

Current 2= Current 3- Current 1

I then plug this into the above equations after I combine common factors and I get:

-1.54(Current 1)+1.8(Current 3- Current 1)=1.4
-1.54(Current 3)-1.8(Current 3- Current 1)=0

I get the Current 3-1 to cancel out, but I'm left with two different currents and I'm stuck. Any idea where I went wrong?
 
Last edited:
  • #4
tmint said:
Thank you.
...

Current 3= Current 1+ Current 2

cabdc: -.77(Current 1)+1.8(Current 2)- 3.5-.77(Current 1)+2.1=0

aefba: -.77(Current 3)-3.5-.77(Current 3)+3.5-1.8(Current 2)

I use the first equation and solve for Current 2 since Current 2 is apart of both equations.

Current 2= Current 3- Current 1

I then plug this into the above equations after I combine common factors and I get:

-1.54(Current 1)+1.8(Current 3- Current 1)=1.4
-1.54(Current 3)-1.8(Current 3- Current 1)=0

I get the Current 3-1 to cancel out, but I'm left with two different currents and I'm stuck. Any idea where I went wrong?
That all looks fine.

(By the way, you have chosen Current 3 to be opposite the direction shown in the figure. No problem however.)

You still have three equation in three unknowns but the last two are now in a convenient form.

Subtract the last two equations. (Bottom from top works nicely.)

What does that give for (Current3 - Current1) ?
 
  • #5
Okay,

1.54[Current 3- Current 1]= 1.4

So [Current 3- Current 1]= 1.1 A

So would Current 2 by that equal 1.1 A?

Right?

Would I then sub this back in for Current 3- Current 1 and solve for Current 1 in the first equation or is that wrong?

Like so,

-1.54[Current 1]+ 1.8[1.1 A] = 1.4 and solve for Current 1?

Current 1 is .376A, that doesn't seem right? I solved this earlier, and really want to understand how its solved. Current 1 came out to .59A not sure exactly how though. My answer before fell within the 5 % tolerance

Did I go about it wrong?


Really appreciate your help.
 
  • #6
tmint said:
Okay,

1.54[Current 3- Current 1]= 1.4
...

Really appreciate your help.
Check your Algebra/Arithmetic .

1.8(Current 3- Current 1) - (-1.8(Current 3- Current 1) ) ≠ 0
 
  • #7
Oh my mistake. So 3.6[Current 3-Current 1]=1.4

Oh you're right, this does work somewhat nicely.

So 3.6[Current 3- Current 1]=1.4

Current 3- Current 1= .389

So Current 2= .389 , I'm kind of suspicious of this value. Its differs from what I got previously. Did I mess up again in my arithmetic again?

and with that value you plug in that for the other ones right into the previous two equations to find Current 1 and Current 3, right?
 
  • #8
tmint said:
Oh my mistake. So 3.6[Current 3-Current 1]=1.4

Oh you're right, this does work somewhat nicely.

So 3.6[Current 3- Current 1]=1.4

Current 3- Current 1= .389

So Current 2= .389 , I'm kind of suspicious of this value. Its differs from what I got previously. Did I mess up again in my arithmetic again?

and with that value you plug in that for the other ones right into the previous two equations to find Current 1 and Current 3, right?
3.6[Current 3- Current 1] is only part of what you get by subtracting those equations.

It's what you left off of the left hand side when you got
1.54[Current 3- Current 1]= 1.4 .​
 
  • #9
Oh okay,

So 1.54(Current 3-Current 1)+3.6(Current 3-Current 1)=1.4

5.14(Current 3-Current 1)=1.4

5.14(Current 2)=1.4

Current 2= .27A!

And I take this value and plug it in back,

I get Current 1= -.593 A change the direction

and I get Current 3= -.323 A change the direction!

And solve for part D which is the EMF of battery 2- (Current 2 times Resistance 2) and I got 3.01 V

Thank you so much! Makes so much more sense now! A big problem of mine was the substituting and your steps helped clear that up. Thanks for not just giving me the solution and helping me figure it out.

I'm actually having another circuit I'm having trouble with, and if you could me that'd be great. I didn't feel creating a new thread for the specific problem was the best way to go about it.

Here is the circuit I'm having trouble with: http://gyazo.com/1e11d9208da0aa29d10b72128844ad40

I set up the following Kirchhoff's equations following the same naming as the last circuit:

cabdc: -217(Current 1) +100(Current 2)+ 6.76=0

aefba: -312(Current 3)-11.8-100(Current 2)=0

Letting Current 2= Current 3-1

Which simplifies to the two equations:

-217(Current 1)+ 100(Current 3-1)=-6.76

-312(Current 3)-100(Current 3-1)=11.8

When I try to simplify this I get

(312-273)(Current 3- Current 1) + 200(Current 3- Current 1)= -18.56

I'm stuck. Could use some help. Don't know if I went about it wrong.
 
Last edited:
  • #10
tmint said:
...
I'm actually having another circuit I'm having trouble with, and if you could me that'd be great. I didn't feel creating a new thread for the specific problem was the best way to go about it.

Here is the circuit I'm having trouble with: http://gyazo.com/1e11d9208da0aa29d10b72128844ad40

I set up the following Kirchhoff's equations following the same naming as the last circuit:

cabdc: -217(Current 1) +100(Current 2)+ 6.76=0

aefba: -312(Current 3)-11.8-100(Current 2)=0

Letting Current 2= Current 3-1

Which simplifies to the two equations:

-217(Current 1)+ 100(Current 3-1)=-6.76

-312(Current 3)-100(Current 3-1)=11.8

When I try to simplify this I get

(312-273)(Current 3- Current 1) + 200(Current 3- Current 1)= -18.56

I'm stuck. Could use some help. Don't know if I went about it wrong.
You really should start a new thread for a new problem.

Now would be a good time to familiarize yourself with the rules & guidelines for Physics Forums.

https://www.physicsforums.com/showthread.php?t=414380

In particular look at Homework Help Guidelines .
 
  • Like
Likes 1 person

1. How do I calculate the total current in a circuit with multiple batteries?

To calculate the total current in a circuit with multiple batteries, you need to first determine the individual currents flowing through each battery. Once you have the individual currents, you can add them together to find the total current. This can be done using Ohm's Law (I=V/R) and Kirchhoff's Circuit Laws.

2. Can the current in a circuit with multiple batteries be different for each battery?

Yes, the current can be different for each battery in a circuit with multiple batteries. This is because the amount of current flowing through each battery is determined by its individual resistance and the voltage across it. So, if the batteries have different resistances or are connected to different components, the current through each one will be different.

3. How does the number of batteries in a circuit affect the total current?

The number of batteries in a circuit does not affect the total current. The total current is determined by the individual currents flowing through each battery, which are in turn determined by the resistance and voltage in the circuit. Adding more batteries in parallel will increase the total voltage, but the total current will remain the same.

4. What happens to the current if one of the batteries in the circuit dies?

If one of the batteries in a circuit dies, the current will decrease. This is because the resistance of the circuit will increase, causing a decrease in the total current. However, if the other batteries in the circuit are still functioning, there will still be some current flowing through the circuit.

5. Can the current in a circuit with multiple batteries be negative?

Yes, the current in a circuit with multiple batteries can be negative. This can happen when the batteries are connected in a way that causes the current to flow in the opposite direction. In this case, the negative current simply indicates the direction of flow in relation to the chosen reference direction.

Similar threads

  • Introductory Physics Homework Help
Replies
3
Views
256
  • Introductory Physics Homework Help
2
Replies
42
Views
1K
  • Introductory Physics Homework Help
Replies
9
Views
393
  • Introductory Physics Homework Help
Replies
2
Views
2K
  • Introductory Physics Homework Help
Replies
1
Views
1K
  • Introductory Physics Homework Help
Replies
4
Views
766
  • Introductory Physics Homework Help
Replies
1
Views
3K
  • Introductory Physics Homework Help
Replies
5
Views
1K
  • Introductory Physics Homework Help
Replies
5
Views
225
  • Introductory Physics Homework Help
Replies
15
Views
1K
Back
Top